Clarification
Why is the answer not D?
Vennela-Vellanki on October 7, 2019
  • December 2004 LSAT
  • SEC2
  • Q6
1
Reply
Why is the answer not a
Why is the answer not a
j906t291 on September 10, 2019
  • December 2004 LSAT
  • SEC2
  • Q13
1
Reply
Question
How do you approach or diagram this question?
Shememories on September 4, 2019
  • December 2004 LSAT
  • SEC2
  • Q7
2
Replies
Working through problem
Can someone please explain this problem which proved difficult? I don't think I could have diagra...
awashing on August 9, 2019
  • December 2004 LSAT
  • SEC2
  • Q16
2
Replies
Type of question
What type of question is this, and how would you approach answering it?
Marissa-Hauck on August 7, 2019
  • December 2004 LSAT
  • SEC2
  • Q9
2
Replies
Answer C
hello! Initially, I chose answer C because I thought it was the better option. Can we elimina...
Julie-V on July 13, 2019
  • December 2004 LSAT
  • SEC2
  • Q6
1
Reply
Answers A and D
Hi LSAT Max! Can someone explain why we can eliminate A? I know that since it's not the right ...
Julie-V on July 12, 2019
  • December 2004 LSAT
  • SEC2
  • Q8
1
Reply
still confused...
I am still confused on why C is the correct answer. I am not sure how to even start this question.
Anna on July 12, 2019
  • December 2004 LSAT
  • SEC2
  • Q15
3
Replies
Question
Why is B incorrect?
Shememories on June 24, 2019
  • December 2004 LSAT
  • SEC2
  • Q13
2
Replies
Explanation
Hi, can you please explain this answer to me? Thank you.
mjenei on June 20, 2019
  • December 2004 LSAT
  • SEC2
  • Q1
1
Reply
Question #1
Hello. Can you please explain the correct answer. I chose D. Thank you
Zahra on June 20, 2019
  • December 2004 LSAT
  • SEC2
  • Q1
8
Replies
Need further Explanation
I think the wording of the passage as a whole confused me. Would you be able to break it down and...
Meckena-Hultin on June 1, 2019
  • December 2004 LSAT
  • SEC2
  • Q23
1
Reply
??
This is extremely confusing. If this is looking for a way to weaken the conclusion then how is th...
Bly2828 on May 13, 2019
  • December 2004 LSAT
  • SEC2
  • Q17
3
Replies
Answer choice A
Why is A not the correct answer choice
zacharydtan on April 29, 2019
  • December 2004 LSAT
  • SEC2
  • Q4
1
Reply
Why not B?
Why is B not a feasible answer, considering the text does state "analysts would stand to gain if ...
santzoulis on March 12, 2019
  • December 2004 LSAT
  • SEC2
  • Q4
3
Replies
Help!
How do you diagram this question? I think the commentator is weakening a cause/effect argument b...
Shememories on March 12, 2019
  • December 2004 LSAT
  • SEC2
  • Q2
4
Replies
How to identify the "argument"
Biologists have noted reproductive abnormalities in fish that are immediately downstream of paper...
SarahA on March 11, 2019
  • December 2004 LSAT
  • SEC2
  • Q11
3
Replies
(D) vs. (E)
The authors conclusion is “But we clearly cannot accept these proclamations of objectivity…” with...
SarahA on March 9, 2019
  • December 2004 LSAT
  • SEC2
  • Q19
3
Replies
How do I avoid over thinking?
Why would it be E, At least he is alive?! Isn't that a "well ending?" I guess I chose B because H...
Derek on February 9, 2019
  • December 2004 LSAT
  • SEC2
  • Q8
4
Replies
Answer B?
Why is the correct answer B, aren't we assuming that Larson would be qualified enough to complete...
SeanF on February 1, 2019
  • December 2004 LSAT
  • SEC2
  • Q3
4
Replies